LSAT and Law School Admissions Forum

Get expert LSAT preparation and law school admissions advice from PowerScore Test Preparation.

User avatar
 Dave Killoran
PowerScore Staff
  • PowerScore Staff
  • Posts: 5852
  • Joined: Mar 25, 2011
|
#78446
Complete Question Explanation
(The complete setup for this game can be found here: lsat/viewtopic.php?t=15592)

The correct answer choice is (B)

The question stem indicates that the computer in office 3 was bought in an earlier year than the printer in office 3. Because the printer in office 3 was purchased in 1988, we can infer that the computer in office 3 was purchased in 1987. In addition, from the third rule we can then infer that the printer in office 4 was purchased in 1987. Finally, from the first rule, we can infer that the computer in office 4 was purchased in 1987:
PT1-Jun1991 game 2 #8 diagram 1.png

Accordingly, answer choice (B) could be true and is correct.
You do not have the required permissions to view the files attached to this post.
 oommenj2
  • Posts: 5
  • Joined: May 09, 2019
|
#75003
Can you explain how we got B for number 8?
User avatar
 KelseyWoods
PowerScore Staff
  • PowerScore Staff
  • Posts: 1079
  • Joined: Jun 26, 2013
|
#75014
Hi oommenj2!

We know from the last rule that the printer in office 3 was bought in 1988. So if the computer in office 3 was bought in an earlier year than the printer in office 3, then the computer in office 3 must have been purchased in 1987. That would mean that the printer in office 4 would have also had to been purchased in 1987 (Rule #3) and the computer in office 4 would, therefore, have to have also been bought in 1987 (Rule #1).

So our local diagram would look like this:

C:                    8           8/9      7           7
P:                      8/9     8/9      8           7
Offices:          1          2          3          4

This is a could be true question, meaning that the 4 incorrect answers will be things that cannot be true and the 1 correct answer will be something that is possible. According to our local diagram, (A), (C), (D), & (E) are all things that cannot be true. The computer in office 2 could be bought in either 1988 or 1989, so answer choice (B) is possible and, therefore, correct.

Hope this helps!

Best,
Kelsey

Get the most out of your LSAT Prep Plus subscription.

Analyze and track your performance with our Testing and Analytics Package.